Đến nội dung

phamngochung9a nội dung

Có 451 mục bởi phamngochung9a (Tìm giới hạn từ 16-05-2020)



Sắp theo                Sắp xếp  

#587870 Topic về Bất đẳng thức, cực trị THCS

Đã gửi bởi phamngochung9a on 07-09-2015 - 21:09 trong Bất đẳng thức và cực trị

Cho $a, b, c > 0$ thoả mãn $(a + b)(b + c)(c + a) = 8$. Tìm GTNN của :

$P = \frac{1}{\sqrt[3]{abc}} + \frac{1}{a + 2b} + \frac{1}{b + 2c} + \frac{1}{c + 2a}$

$(2a+2b)(2b+2c)(2c+2a)=64\Leftrightarrow 64\geq \left ( \sqrt[3]{abc}+\sqrt[3]{(a+2b)(b+2c)(c+2a)} \right )^{3}\Rightarrow \sqrt[3]{abc}+\sqrt[3]{(a+2b)(b+2c)(c+2a)}\leq 4$

Vậy $P\geq \frac{1}{\sqrt[3]{abc}}+\frac{3}{\sqrt[3]{(a+2b)(b+2c)(c+2a)}}= \frac{\frac{1}{3}}{\sqrt[3]{abc}}+\frac{3}{\sqrt[3]{(a+2b)(b+2c)(c+2a)}}+\frac{\frac{2}{3}}{\sqrt[3]{abc}}\geq \frac{\left ( \sqrt{\frac{1}{3}}+\sqrt{3} \right )^{2}}{4}+\frac{\frac{2}{3}}{1}=2$




#553132 Violympic 2015

Đã gửi bởi phamngochung9a on 11-04-2015 - 10:58 trong Góc giao lưu

Đắng lòng :(  :( .....!!!!!!!, năm nay Thanh Hóa ko tổ chức thi violympic  :icon2:




#638074 Tổng hợp các bài BĐT trong các đề thi thử THPT Quốc Gia môn Toán năm 2017

Đã gửi bởi phamngochung9a on 04-06-2016 - 20:48 trong Bất đẳng thức và cực trị

Cuối cùng thì file pdf của topic đã hoàn thành  :D

 

File gửi kèm  batdangthucvmf (11).pdf   182.91K   234 Số lần tải

 

File sẽ được cập nhật liên tục...

 

 

 

 




#637696 Tổng hợp các bài BĐT trong các đề thi thử THPT Quốc Gia môn Toán năm 2017

Đã gửi bởi phamngochung9a on 02-06-2016 - 21:43 trong Bất đẳng thức và cực trị

Hiện tại mình đang tổng hợp file pdf của topic. Các bạn cho ý kiến về nó nhé

Liệu có cần đáp án không nhỉ ?

File gửi kèm




#635374 Tổng hợp các bài BĐT trong các đề thi thử THPT Quốc Gia môn Toán năm 2017

Đã gửi bởi phamngochung9a on 25-05-2016 - 09:59 trong Bất đẳng thức và cực trị

chỗ$(ab+bc+ca)-\frac{(ab+bc+ca)^{2}}{3}\geq 0$ với $ab+bc+ca\leq9$

sai ở chỗ là chưa biết dấu của $(a+b+c)^{2}-abc(a+b+c)$

Ui, tôi nhầm tí, bạn giải đúng rồi đó:

Ta chứng minh được 

$\left ( a+b+c \right )^{2}-abc\left ( a+b+c \right )\geq 3\left ( ab+bc+ca \right )-\frac{\left ( ab+bc+ca \right )^{2}}{3}\geq 0$

thì hiển nhiên $(a+b+c)^{2}-abc(a+b+c)\geq 0$  rồi




#640121 Tổng hợp các bài BĐT trong các đề thi thử THPT Quốc Gia môn Toán năm 2017

Đã gửi bởi phamngochung9a on 13-06-2016 - 20:57 trong Bất đẳng thức và cực trị

Bài 155:  $ Cho\quad x,y,z>0\quad thõa\quad mãn\quad :\quad { x }^{ 2 }+{ 8y }^{ 2 }+{ 8z }^{ 2 }=20.\quad Tìm\quad GTLN:\\ P=\frac { { 2x }^{ 2 } }{ { x }^{ 2 }(4-yz)+8 } +\frac { y+z }{ x+y+z+1 } -\frac { { x }^{ 2 }+{ (y+z) }^{ 2 } }{ 100 }  $

Ta sẽ chứng minh:

$x^{2}\left ( 4-yz \right )+8\geq 2x\left ( x+y+z+1 \right )$

 

Thật vậy:

 

Xét hiệu

 

$x^{2}\left ( 4-yz \right )+8- 2x\left ( x+y+z+1 \right )\\=2x^{2}-2x-x^{2}.yz-2x\left ( y+z \right )+8\\\geq 2x^{2}-2x+8-\frac{1}{2}x^{2}\left ( y^{2}+z^{2} \right )-x^{2}-\left ( y+z \right )^{2}\\\geq 2x^{2}-2x+8-\frac{1}{2}x^{2}.\frac{20-x^{2}}{8}-x^{2}-\frac{20-x^{2}}{4}\\=\frac{1}{16}\left ( x^{4}-32x+48 \right )\\=\frac{1}{16}\left [ \left ( x^{4}+16+16+16 \right )-32x \right ]\\\geq \frac{1}{16}.\left ( 32x-32x \right )=0$

 

Vậy:

$P\leq \frac{x}{x+y+z+1}+\frac{y+z}{x+y+z+1}-\frac{\left ( x+y+z \right )^{2}}{200}=\frac{x+y+z}{x+y+z+1}-\frac{\left ( x+y+z \right )^{2}}{200}$

 

Đặt $x+y+z=t$ và khảo sát hàm số là xong




#637663 Tổng hợp các bài BĐT trong các đề thi thử THPT Quốc Gia môn Toán năm 2017

Đã gửi bởi phamngochung9a on 02-06-2016 - 19:47 trong Bất đẳng thức và cực trị

 

Bài 128 (Đinh Xuân Hùng)

 

Cho $x,y,z$ là các số thực dương thỏa mãn : $xyz+4\geq 2(x+y+z)$ . Tìm giá trị lớn nhất của biểu thức : 

 

$$\frac{\sqrt{2x^2-4x+4}+\sqrt{2y^2-4y+4}+\sqrt{2z^2-4z+4}}{2(x+y+z)^2}+\frac{9}{xyz+4}$$

 

Ta có:

$VT=P\leq \frac{\sqrt{6\left ( x^{2}+y^{2}+z^{2}-2x-2y-2z+6 \right )}}{2\left ( x+y+z \right )^{2}}+\frac{9}{2\left ( x+y+z \right )}\\=\frac{\sqrt{6\left [ \left ( x+y+z \right )^{2}-2\left ( x+y+z \right )-2\left ( xy+yz+zx \right )+6 \right ]}}{\left ( x+y+z \right )^{2}}$

Từ giả thiết, ta có:

$\sqrt{\left ( \frac{xy+yz+zx}{3} \right )^{3}}+4\geq 2\sqrt{3\left ( xy+yz+zx \right )}\\\Leftrightarrow xy+yz+zx\geq 12$

Vậy:

$P\leq \frac{\sqrt{6\left [ \left ( x+y+z \right )^{2}-2\left ( x+y+z \right )-18 \right ]}}{2\left ( x+y+z \right )^{2}}+\frac{9}{2\left ( x+y+z \right )}$

Đặt $t=x+y+z$, ta có: $t\in \left [ 6;+\infty \right )$

 

Khi đó: $P\leq \frac{\sqrt{6\left ( t^{2}-2t-18 \right )}}{2t^{2}}+\frac{9}{2t}$

 

Khảo sát hàm số trên với $t\in \left [ 6;+\infty \right )$, ta được:

 

$\max P=\frac{5}{6}\Leftrightarrow x=y=z=2$




#640566 Tổng hợp các bài BĐT trong các đề thi thử THPT Quốc Gia môn Toán năm 2017

Đã gửi bởi phamngochung9a on 15-06-2016 - 20:14 trong Bất đẳng thức và cực trị

Hình như bác tính f'(z) sai

Ờ, tại lúc đó vội quá nên hấp tấp  :D

Em xin giải lại:

 

$P\leq \left ( 1-y^{2} \right )\left ( y-z \right )+z^{2}\left ( 1-z \right )$

 

Coi biểu thức trên là hàm biến $y$, ta có:

 

$f'\left ( y \right )=-3y^{2}+2zy+1=0\\\Rightarrow y=\frac{\sqrt{z^{2}+3}+z}{3}$

 

$\Rightarrow f\left ( y \right )\leq f\left ( \frac{z+\sqrt{z^{2}+3}}{3} \right )\\=\frac{1}{27}\left ( 6\sqrt{z^{2}+3}+2z^{2}\sqrt{z^{2}+3}+2z^{3}-18z \right )+z^{2}\left ( 1-z \right )\\\leq \frac{1}{27}\left ( 6\sqrt{z^{2}+3}+4z^{2}+2z^{3}-18z+27z^{2}-27z^{3} \right )\\=\frac{1}{27}\left ( 6\sqrt{z^{2}+3}+31z^{2}-25z^{3}-18z \right )\\\leq \frac{1}{27}\left ( 6\sqrt{z^{2}+3}+31z^{2}-2\sqrt{25.18}.z^{2} \right )\\\leq \frac{1}{27}\left ( 6\sqrt{z^{2}+3} -11z^{2}\right )$

 

Đến đây đặt $t=\sqrt{z^{2}+3} \quad \quad \left ( t\in \left [ \sqrt{3};2 \right ] \right )$

và khảo sát hàm chắc là ngon ròi

 

Bác kiểm tra xem có tính sai chỗ nào không hộ em nhá

 

Mở rộng:  Với cách làm trên, ta hoàn toàn không hề sử dụng đến giả thiết $z\leq y\leq x$, vì vậy ta có thể chứng minh bài toán chặt hơn là:

 

Với mọi $x,y,z\in \left [ 0;1 \right ]$ thì:

 
 

$\left ( x^{2}-y^{2} \right )\left ( y-z \right )+z^{2}\left ( 1-z \right )\leq \frac{2\sqrt{3}}{9}$




#642233 Tổng hợp các bài BĐT trong các đề thi thử THPT Quốc Gia môn Toán năm 2017

Đã gửi bởi phamngochung9a on 26-06-2016 - 10:03 trong Bất đẳng thức và cực trị

Bài 185: Cho $a,b,c$ là độ dài 3 cạnh của một tam giác và thỏa mãn $a^2+b^2+c^2+ab-2bc+2ca=0$.

Tìm GTNN của $P=\frac{c^2}{(a+b-c)^2}+\frac{c^2}{a^2+b^2}+\frac{\sqrt{ab}}{a+b}$ 

Đúng như bạn Caobo171 đã nói, bài này cũng sai đề. Đây là câu cuối đề thi HSG Bắc Giang lớp 12 năm 2015- 2016. 

Giả thiết đúng phải là: $a^2+b^2+c^2+ab-2bc{\color{Red}- }2ca=0$

 

Đặt $\left\{\begin{matrix} a=xc & \\ b=yc & \end{matrix}\right.$ khi đó: $x^{2}+y^{2}+xy+1-2x-2y=0$ 

 

Khi đó: $a+b> c\Rightarrow xc+yc> c\Rightarrow x+y> 1$

 

Ta có:

 

$P=\frac{1}{\left ( x+y-1 \right )^{2}}+\frac{1}{x^{2}+y^{2}}+\frac{\sqrt{xy}}{x+y}$

 

Biến đổi giả thiết:

 

  • $xy=\left ( x+y-1 \right )^{2}\Rightarrow \sqrt{xy}=x+y-1$  (do $x+y> 1$ )

 

  • $x^{2}+y^{2}=-\left ( x+y \right )^{2}+4\left ( x+y \right )-2$

Vậy:

 

$P=\frac{1}{\left ( x+y-1 \right )^{2}}+\frac{1}{-\left ( x+y \right )^{2}+4\left ( x+y \right )-2}+\frac{x+y-1}{x+y}$

 

Đặt $x+y=t$ và khảo sát hàm chắc là ngon rồi  :D




#642228 Tổng hợp các bài BĐT trong các đề thi thử THPT Quốc Gia môn Toán năm 2017

Đã gửi bởi phamngochung9a on 26-06-2016 - 09:43 trong Bất đẳng thức và cực trị

Bài 181: Cho $a,b,c$ là các số thực thỏa mãn: $a,b,c\in (0,1)$ và $ab+bc+ca=1$.

Tìm GTNN của biểu thức: $P=\sum \frac{a^2(1-2b)}{b}$

Bài 181:

 

$P=\sum \frac{a^{2}\left ( 1-b \right )}{b}-\left ( a^{2}+b^{2}+c \right )$

 

Vì $a,b,c\in \left ( 0;1 \right )$ nên ta sử dụng AM-GM cho cặp số dương :

$\frac{a^{2}\left ( 1-b \right )}{b}+b\left ( 1-b \right )\geq 2a\left ( 1-b \right )\\\Rightarrow \frac{a^{2}\left ( 1-b \right )}{b}\geq 2a-b+b^{2}-2ab$

 

Xây dựng các BĐT tương tự rồi cộng lại, ta được:

 

$\sum \frac{a^{2}\left ( 1-b \right )}{b}\geq a+b+c+a^{2}+b^{2}+c^{2}-2\\\Rightarrow \sum \frac{a^{2}\left ( 1-2b \right )}{b}\geq a+b+c-2\geq \sqrt{3\left ( ab+bc+ca \right )}-2=\sqrt{3}-2$

 

Vậy $\min P=\sqrt{3}-2\Leftrightarrow a=b=c=\frac{1}{\sqrt{3}}$




#642225 Tổng hợp các bài BĐT trong các đề thi thử THPT Quốc Gia môn Toán năm 2017

Đã gửi bởi phamngochung9a on 26-06-2016 - 09:31 trong Bất đẳng thức và cực trị

Khuấy động topic nào mọi người:

 

Bài 186: Cho các số thực dương $x,y,z$ thỏa mãn $x\geq 2$ và $x+y+z=6$. Tìm giá trị lớn nhất của biểu thức:

 

$$P=\frac{\left ( y+z-2 \right )\left ( xy\sqrt{9y+3z}+y^{2}\sqrt{9x-3z} \right )+yz\left ( \sqrt{y}-\sqrt{x} \right )}{y\left ( y+\sqrt{x} \right )\left ( \sqrt{x}+1 \right )+1}+\frac{3z\sqrt{2}}{4}$$




#640673 Tổng hợp các bài BĐT trong các đề thi thử THPT Quốc Gia môn Toán năm 2017

Đã gửi bởi phamngochung9a on 16-06-2016 - 12:18 trong Bất đẳng thức và cực trị

Bài 162 (Đề thi thử  THPT Chuyên Lương Văn Tụy lần 3)

 

Cho các số thực dương $x,y,z$ thỏa mãn $x>y>z>0$ . Tìm giá trị nhỏ nhất của biểu thức:

 

$$P=\frac{y}{x-y}+\frac{z}{y-z}+\frac{x^2}{8z\left ( \sqrt{xz}-z \right )}$$

 

Áp dụng " Bá đạo thức " Cauchy- Schwarz, ta có:

 

$P=\frac{x}{x-y}+\frac{z}{y-z}+\frac{x^{2}}{8z\left ( \sqrt{xz}-z \right )}-1\\\geq \frac{\left ( \sqrt{x}+\sqrt{z} \right )^{2}}{x-z}+\frac{x^{2}}{8z\left ( \sqrt{xz}-z \right )}-1$

 

Đặt $t=\sqrt{\frac{x}{z}}$, khi đó:

 

$P\geq \frac{\left ( t+1 \right )^{2}}{t^{2}-1}+\frac{t^{4}}{8\left ( t-1 \right )}-1\\=\frac{t^{4}+8t+8}{8\left ( t-1 \right )}-1\\=\frac{t^{4}-32t+48}{8\left ( t-1 \right )}+4\\=\frac{t^{4}+16+16+16-32t}{8\left ( t-1 \right )}+4\\\geq \frac{4\sqrt[4]{t^{4}.16^{3}}-32}{8\left ( t-1 \right )}+4\geq 4$
 
Vậy $\min P=4\Leftrightarrow x=2y=4z$



#635367 Tổng hợp các bài BĐT trong các đề thi thử THPT Quốc Gia môn Toán năm 2017

Đã gửi bởi phamngochung9a on 25-05-2016 - 09:26 trong Bất đẳng thức và cực trị

đầu tiên chúng ta dễ dàng chứng minh bổ đề quen thuộc $(a^{2}+2)(b^{2}+2)(c^{2}+2)\geq 3(a+b+c)^{2}$

Nếu $ab+bc+ca\geq 9$ thì $6(a+b+c)^{2}\geq 18(ab+bc+ca)\geq 18.9=162>81$

Nếu $ab+bc+ca\leq 9$ thì ta có $abc(a+b+c)\leq \frac{(ab+bc+ca)^{2}}{3}$ ta có

$P\geq \frac{3\left [ (a+b+c)^{2}-abc(a+b+c) \right ]^{2}}{(ab+bc+ca-1)^{2}}+6(a+b+c)^{2}\geq \frac{\left [ 9(ab+bc+ca)-(ab+bc+ca)^{2} \right ]^{2}}{3(ab+bc+ca-1)^{2}}+18(ab+bc+ca)$.

từ giả thuyết ta dễ dàng suy ra được $abc\geq 1\Rightarrow ab+bc+ca\geq 3$.Đặt t=ab+bc+ca

đến đây xét hàm là xong

MinP=81 đẳng thức xảy ra khi a=b=c=1

Lúc đầu tui cũng giải cách ni nè, nhưng cách ni sai rùi

Chỗ 

$\left ( a+b+c \right )^{2}-abc\left ( a+b+c \right )\geq 3\left ( ab+bc+ca \right )-\frac{\left ( ab+bc+ca \right )^{2}}{3}$

Ta không thể suy ra được rằng:

$\left [ \left ( a+b+c \right )^{2}-abc\left ( a+b+c \right ) \right ]^{2}\geq \left [ 3\left ( ab+bc+ca \right )-\frac{\left ( ab+bc+ca \right )^{2}}{3} \right ]^{2}$

vì ta chưa biết dấu của $3\left ( ab+bc+ca \right )-\frac{\left ( ab+bc+ca \right )^{2}}{3}$

 

Một ví dụ đơn giản. Chẳng hạn 

$5>-9\Rightarrow 5^{2}> \left ( -9 \right )^{2}???$

Nếu muốn bình phương được, ta phải chứng minh cho 

$3\left ( ab+bc+ca \right )-\frac{\left ( ab+bc+ca \right )^{2}}{3}>0\\\Leftrightarrow ab+bc+ca< 9$

Điều này không hề đúng 




#635245 Tổng hợp các bài BĐT trong các đề thi thử THPT Quốc Gia môn Toán năm 2017

Đã gửi bởi phamngochung9a on 24-05-2016 - 19:53 trong Bất đẳng thức và cực trị

Bài 104:Cho $a,b,c$ là những số thực dương thỏa mãn $0< \frac{ab+bc+ca-abc}{ab+bc+ca-1}\leq 1$. 

Tìm giá trị nhỏ nhất của biểu thức:

   $P=\left ( a^{2}+2 \right )\left ( b^{2}+2 \right )\left ( c^{2}+2 \right )\left [ \left ( \frac{a+b+c-abc}{ab+bc+ca-1} \right )^{2}+2 \right ]$




#627665 Tổng hợp các bài BĐT trong các đề thi thử THPT Quốc Gia môn Toán năm 2017

Đã gửi bởi phamngochung9a on 17-04-2016 - 09:15 trong Bất đẳng thức và cực trị

Bài 30: (THPT chuyên Thái Bình- lần 3)

Cho ba số thực dương $x,y,z$ thỏa mãn $x+2y+3z=1$. Tìm giá trị lớn nhất của biểu thức:

$P=x^{2}\left ( 5-6x \right )+4y^{2}\left ( 5-12y \right )+z^{2}\left ( 45-162z \right )$

Đặt $\left\{\begin{matrix} x=a & & \\ y=2b & & \\ z=3c & & \end{matrix}\right.$

Dễ nhận thấy BĐT đạt điểm rơi tại $a=b=c=\frac{1}{3}$ hoặc $a=b=\frac{1}{2};c=0$ điều này khiến ta liên tưởng đến BĐT Schur.

Đồng bậc hai vế, ta cần chứng minh:

$5\left ( a+b+c \right )\left (a^{2}+b^{2}+c^{2} \right )-6\left ( a^{3}+b^{3}+c^{3} \right )\leq \left ( a+b+c \right )^{3}\\\Leftrightarrow a^{3}+b^{3}+c^{3}+3abc\geq ab\left ( a+b \right )+bc\left ( b+c \right )+ca\left ( c+a \right )$

Vì đây là đề thi THPT QG nên sẽ phải cm lại BĐT Schur. Cách chứng minh ngắn nhất là sử dụng tính chất đối xứng.

Gs $a\geq b\geq c\geq 0$, ta có:

$a^{3}+b^{3}+c^{3}+3abc-ab\left ( a+b \right )-bc\left ( b+c \right )-ca\left ( c+a \right )\\=a\left ( a-b \right )\left ( a-c \right )+b\left ( b-a \right )\left ( b-c \right )+c\left ( c-a \right )\left ( c-b \right )\\\geq a\left ( a-b \right )\left ( b-c \right )+b\left ( b-a \right )\left ( b-c \right )+c\left ( a-c \right )\left ( b-c \right )\\=\left ( a-b \right )^{2}\left ( b-c \right )+c\left ( a-c \right )\left ( b-c \right )\geq 0\left ( TRUE \right )$

Vậy $max P= 1$ khi........ copy ở trên




#627660 Tổng hợp các bài BĐT trong các đề thi thử THPT Quốc Gia môn Toán năm 2017

Đã gửi bởi phamngochung9a on 17-04-2016 - 08:48 trong Bất đẳng thức và cực trị

Cho mình hỏi tại sao lại giả sử y nằm giữa x và z vậy ? Vì mình thấy đa số bài toán đều giả sử y= min{x,y,z} hoặc y= max{x,y,z} khi các biến hoán vị  :mellow:

Đối với các bài toán hoán vị, ta được phép giả sử vị trí của một biến bất kì, chứ không bắt buộc phải giả sử là lớn nhất hay nhỏ nhất đâu bạn. Ở đây, gs lớn nhất là vị trí đầu, gs nhỏ nhất là vị trí cuối.

 

Lời giải :

 

Đặt $a=x,b=2y,c=3z$ thì $a+b+c=4$. Khi đó :

$$P=a^2(5-6a)+b^2(5-6b)+c^2(5-6c)=5a^2+5b^2+5c^2-6a^3-6b^3-6c^3$$

Bằng phương pháp tiếp tuyến, chỉ ra được :

$$5a^2-6a^3\leq \dfrac{-56}{3}a+\dfrac{176}{9}$$

Suy ra :

$$P\leq \dfrac{-56}{3}(a+b+c)+\dfrac{176}{3}=-16$$

$$MaxP=-16\Leftrightarrow x=\dfrac{4}{3},y=\dfrac{2}{3},z=\dfrac{4}{9}$$

Xin lỗi anh nha, tại em hấp tấp gõ sai đề. Lúc đầu em cũng nghĩ dùng UTC nhưng sau đó thất bại, thấy anh làm PP này lại được kiểm tra lại đề thì.....Bây giờ thì không dùng UTC được đâu anh  :P (em nghĩ vậy)




#627372 Tổng hợp các bài BĐT trong các đề thi thử THPT Quốc Gia môn Toán năm 2017

Đã gửi bởi phamngochung9a on 15-04-2016 - 23:12 trong Bất đẳng thức và cực trị

Bài 28: (Đề thi thử của sở GD- ĐT Thanh Hóa năm 2016)

Cho $a,b,c$ là độ dài ba cạnh của một tam giác. Tìm giá trị lớn nhất của biểu thức:

$P=\left ( a+b+c \right )\left ( \frac{3a-b}{a^{2}+ab}+\frac{3b-c}{b^{2}+bc}+\frac{3c-a}{c^{2}+ca} \right )$

 

Bài 29: (Đề thi thử của sở GD- ĐT Bắc Giang năm 2016)

Cho ba số thực dương $x,y,z$ thỏa mãn $xy+yz+zx+xyz=4$. Chứng minh rằng:

$3\left ( \frac{1}{\sqrt{x}}+\frac{1}{\sqrt{y}}+\frac{1}{\sqrt{z}} \right )^{2}\geq \left ( x+2 \right )\left ( y+2 \right )\left ( z+2 \right )$

 

Bài 30: (THPT chuyên Thái Bình- lần 3)

Cho ba số thực dương $x,y,z$ thỏa mãn $x+2y+3z=1$. Tìm giá trị lớn nhất của biểu thức:

$P=x^{2}\left ( 5-6x \right )+4y^{2}\left ( 5-12y \right )+z^{2}\left ( 45-162z \right )$




#624384 Tổng hợp các bài BĐT trong các đề thi thử THPT Quốc Gia môn Toán năm 2017

Đã gửi bởi phamngochung9a on 02-04-2016 - 22:40 trong Bất đẳng thức và cực trị

$\boxed{21}$ (Đề thi thử môn Toán lần 3/2016 trường THPT Chuyên Khoa học tự nhiên)

 

Cho $a,b,c$ là các số thực không âm thỏa mãn $a^2+ab+b^2=c(a+b+c)$ .Tìm giá trị lớn nhất của biểu thức:

 

$$P=\frac{(a+c)^2}{2a^2+2ac+c^2}+\frac{(b+c)^2}{2b^2+2bc+c^2}+\frac{ab}{(a+b)^2}+\frac{ab}{a^2+4ab+b^2}$$

Thực chất bài này là sử dụng bổ đề 

$\frac{1}{1+x^{2}}+\frac{1}{1+y^{2}}\leq \frac{2}{1+xy}$ với $xy\leq 1$

 

Biến đổi giả thiết thành $(c+a)(c+b)=(a+b)^{2}$

Ta có:

$P= \frac{1}{\left ( \frac{a}{a+c} \right )^{2}+1}+\frac{1}{\left ( \frac{b}{b+c} \right )^{2}+1}+\frac{ab}{(a+b)^{2}}+\frac{1}{2+\frac{(a+b)^{2}}{ab}}$

Do $\frac{a}{a+c}.\frac{b}{b+c}=\frac{ab}{(a+b)^{2}}\leq \frac{1}{4}< 1$ nên

 

$P\leq \frac{2}{1+\frac{ab}{(a+c)(b+c)}}+\frac{ab}{(a+b)^{2}}+\frac{1}{2+\frac{(a+b)^{2}}{ab}}\\=\frac{2}{1+\frac{ab}{(a+b)^{2}}}+\frac{ab}{(a+b)^{2}}+\frac{1}{2+\frac{(a+b)^{2}}{ab}}$

Đặt $\frac{ab}{(a+b)^{2}}=t$  ( với $t\leq \frac{1}{4}$ )

$\Rightarrow P=\frac{2}{1+t}+t+\frac{1}{2+\frac{1}{t}}\\=\frac{2}{1+t}+t+\frac{t}{2t+1}$

Khảo sát hàm số $f(t)=\frac{2}{1+t}+t+\frac{t}{2t+1}$ trên $\left (0;\frac{1}{4} \right ]$, ta có:

$f(t)\leq f\left ( \frac{1}{4} \right )=\frac{121}{60}$

Vậy $\max P=\frac{121}{60}\Leftrightarrow ...........$




#631424 Tổng hợp các bài BĐT trong các đề thi thử THPT Quốc Gia môn Toán năm 2017

Đã gửi bởi phamngochung9a on 05-05-2016 - 20:01 trong Bất đẳng thức và cực trị

Ta sẽ chứng minh 2 BĐT sau

 

$\left\{\begin{matrix} \frac{1}{\left ( a+c \right )^2}\geq b^2 & \\ \frac{1}{\left ( b+c \right )^2}\geq a^2 & \end{matrix}\right.$

 

Chứng minh BĐT  đầu ,BĐT sau tương tự

 

$\frac{1}{\left ( a+c \right )^2}\geq b^2 \Leftrightarrow \frac{\left ( ab+bc+ca \right )^2}{\left ( a+c \right )^2}-b^2\geq 0$

 

$\Leftrightarrow \frac{ac \left ( 2ab+2bc+ac \right )}{\left ( a+c \right )^2}\geq 0$

 

BĐT trên đúng. Vậy BĐT phụ được chứng minh

 

Suy ra

 

$P\geq \frac{8}{\left ( a+b \right )^2}+3a^2+3b^2\geq \frac{8}{\left ( a+b \right )^2}+\frac{3\left ( a+b \right )^2}{2}\geq 4\sqrt{3}$

 

Đẳng thức xảy ra $\Leftrightarrow \left\{\begin{matrix} c=0 & \\ a=b=\frac{2}{\sqrt{3}} & \end{matrix}\right.$

 

Vậy $MinP=4\sqrt{3}\Leftrightarrow \Leftrightarrow \left\{\begin{matrix} c=0 & \\ a=b=\frac{2}{\sqrt{3}} & \end{matrix}\right.$

 

 

Mong các bạn không ra thêm bài làm loãng topic.
Còn rất nhiều bài chưa làm ở trên

Không biết có spam hay không nhưng hình như 

$\left\{\begin{matrix} c=0 & \\ a=b=\frac{2}{\sqrt{3}} & \end{matrix}\right.$

 không thỏa mãn điều kiện giả thiết, bạn ạ

Giả thiết là $ab+bc+ca=1$ mà




#631431 Tổng hợp các bài BĐT trong các đề thi thử THPT Quốc Gia môn Toán năm 2017

Đã gửi bởi phamngochung9a on 05-05-2016 - 20:21 trong Bất đẳng thức và cực trị

Bài 62: Cho x,y,z thuộc [1;2]. Tìm giá trị lớn nhất của biểu thức:

$E=\frac{2(xy+yz+zx)}{xyz+2(2x+y+z)}+\frac{8}{2x(y+z)+yz+4}-\frac{y+z+4}{\sqrt{yz}+1}$

$\left ( y-2 \right )\left ( z-2 \right )\geq 0\\\Rightarrow yz+4\geq 2y+2z$

  • $xyz+2\left ( 2x+y+z \right )=x\left ( yz+4 \right )+2y+2z\geq x.\left ( 2y+2z \right )+2\left ( y+z \right )=2\left ( x+1 \right )\left ( y+z \right )$
  • $2x\left ( y+z \right )+yz+4\geq 2x\left ( y+z \right )+2y+2z=2\left ( x+1 \right )\left ( y+z \right )$

Vậy: $E\leq \frac{xy+yz+zx+4}{\left ( x+1 \right )\left ( y+z \right )}-\frac{y+z+4}{\sqrt{yz}+1}\\\leq 1-\frac{1}{x+1}+\frac{\frac{\left ( y+z \right )^{2}}{4}+4}{2\left ( y+z \right )}-\frac{y+z+4}{\frac{y+z}{2}+1}\\\leq \frac{1}{2}+\frac{\left ( y+z \right )^{2}+16}{8\left ( y+z \right )}-\frac{2\left ( y+z \right )+8}{y+z+2}$

Đặt $y+z=t$, khi đó: $t\in \left [ 2;4 \right ]$ và:

$E\leq \frac{1}{2}+\frac{t^{2}+16}{8t}-\frac{2t+8}{t+2}$

Khảo sát hàm số trên với $t\in \left [ 2;4 \right ]$, ta được: $E\leq -\frac{7}{6}$




#635056 Tổng hợp các bài BĐT trong các đề thi thử THPT Quốc Gia môn Toán năm 2017

Đã gửi bởi phamngochung9a on 23-05-2016 - 21:21 trong Bất đẳng thức và cực trị

Mình cũng xin đóng góp một số bài

Bài 93:cho a,b,c là các số thực dương thỏa mãn abc=1.CMR

$\frac{1}{(a+1)^{2}}+\frac{1}{(1+b)^{2}}+\frac{1}{(1+c)^{2}}+\frac{4}{(1+a+b+c)^{2}}\geq 1$

Chắc chỉ có cách trâu bò ni mới xơi được:

Đặt $\left\{\begin{matrix} \frac{1}{a+1}=x & & \\ \frac{1}{b+1}=y & & \\ \frac{1}{c+1}=z & & \end{matrix}\right.$

Ta có: 

$abc=\left ( \frac{1}{x}-1 \right )\left ( \frac{1}{y}-1 \right )\left ( \frac{1}{z}-1 \right )=1\\\Leftrightarrow 2xyz=1-x-y-z+xy+yz+zx$

Khi đó:

$VT=x^{2}+y^{2}+z^{2}+\frac{\left ( 1+xy+yz+zx-x-y-z \right )^{2}}{\left ( x+y+z-1 \right )^{2}}$

Đặt $\left\{\begin{matrix} p=x+y+z & \\ q=xy+yz+zx & \end{matrix}\right.$, ta cần chứng minh:

$p^{2}-2q+\frac{\left ( 1+q-p \right )^{2}}{\left ( p-1 \right )^{2}}\geq 1\\\Leftrightarrow p^{2}\left ( p-1 \right )^{2}-2q\left (p-1 \right )^{2}+1+p^{2}+q^{2}+2q-2pq-2p\geq p^{2}-2p+1\\\Leftrightarrow p^{2}\left ( p-1 \right )^{2}-2p^{2}q+4pq-2q+q^{2}+2q-2pq\geq 0\\\Leftrightarrow p^{2}\left ( p-1 \right )^{2}-2p\left ( p-1 \right ).q+q^{2}\geq 0\\\Leftrightarrow \left ( p^{2}-p-q \right )^{2}\geq 0\left ( TRUE \right )$




#632301 Tổng hợp các bài BĐT trong các đề thi thử THPT Quốc Gia môn Toán năm 2017

Đã gửi bởi phamngochung9a on 10-05-2016 - 19:31 trong Bất đẳng thức và cực trị

Bài 82: (Chuyên Đại học Vinh lần 3 năm 2016 )

Cho $x,y,z$ là những số thực thuộc khoảng $\left ( 1;4 \right )$. Tìm giá trị lớn nhất của biểu thức:

$P=2.\frac{\left ( y+z-x \right )^{2}}{yz}+\sqrt{3}.\frac{\left ( z+x-y \right )^{2}}{zx}-2\sqrt{3}.\frac{\left ( x+y-z \right )^{2}}{xy}$




#632137 Tổng hợp các bài BĐT trong các đề thi thử THPT Quốc Gia môn Toán năm 2017

Đã gửi bởi phamngochung9a on 09-05-2016 - 20:53 trong Bất đẳng thức và cực trị

Oops. Cảm ơn bạn nhé :)) Mình lộn mất rồi. Mà nếu theo lời người ra bài thì số 3 phải thay bằng số 1 thì ms chuẩn đề thầy Nam :V Nếu nt thì dấu bằng sẽ là c=0,a=b=1. Mình đoán là có nhầm lẫn :P

Đề bài không sai đâu bạn. Sau nhiều thời gian suy nghĩ, mình có lời giải này, bạn xem hộ mình nhé.  :icon6:

 

$P=\frac{8}{\left ( a+b \right )^{2}}+\frac{3}{\left ( b+c \right )^{2}}+\frac{3}{\left ( c+a \right )^{2}}\\=\frac{8}{\left ( a+b \right )^{2}}+3.\frac{\left ( ab+bc+ca \right )^{2}}{\left ( b+c \right )^{2}}+3.\frac{\left ( ab+bc+ca \right )^{2}}{\left ( c+a \right )^{2}}\\=\frac{8}{\left ( a+b \right )^{2}}+3a^{2}+3b^{2}+6abc\left ( \frac{1}{b+c}+\frac{1}{c+a} \right )+3.\frac{b^{2}c^{2}}{\left ( b+c \right )^{2}}+3.\frac{c^{2}a^{2}}{\left ( c+a \right )^{2}}\\\geq \frac{8}{\left ( a+b \right )^{2}}+3\left ( a^{2}+b^{2} \right )+6abc.\frac{2c+a+b}{c^{2}+1}\\\geq \frac{8}{\left ( a+b \right )^{2}}+3\left ( a^{2}+b^{2} \right )+6abc.\frac{c^{2}\left ( a+b \right )+a+b}{c^{2}+1}\\=\frac{8}{\left ( a+b \right )^{2}}+3\left ( a^{2}+b^{2} \right )+6abc\left ( a+b \right )=\frac{8}{\left ( a+b \right )^{2}}+3\left ( a^{2}+b^{2} \right )+6ab\left ( 1-ab \right )\\=\frac{8}{\left ( a+b \right )^{2}}+3\left ( a+b \right )^{2}-6a^{2}b^{2}$

Đặt $a+b=t$

  • Nếu $a+b\geq 2$, ta có:

$P\geq \frac{8}{\left ( a+b \right )^{2}}+3\left ( a+b \right )^{2}-6=\frac{8}{t^{2}}+3t^{2}-6\geq 8$

  • Nếu $a+b\leq 2$, ta có:

$P\geq \frac{8}{t^{2}}+3t^{2}-\frac{3}{8}t^{4}\geq 8$

 

Vậy $P_{\min}=1\Leftrightarrow \left\{\begin{matrix} a=b=1 & \\ c=0 & \end{matrix}\right.$




#631459 Tổng hợp các bài BĐT trong các đề thi thử THPT Quốc Gia môn Toán năm 2017

Đã gửi bởi phamngochung9a on 05-05-2016 - 21:04 trong Bất đẳng thức và cực trị

Bài 63:Cho x,y,z thuộc [1;2]. Tìm giá trị lớn nhất của:
$F=(x+y+z)(\frac{1}{x}+\frac{1}{y}+\frac{1}{z})$

$F=3+\frac{x}{y}+\frac{y}{z}+\frac{z}{x}+\frac{y}{x}+\frac{z}{y}+\frac{x}{z}$

Không mất tính tổng quát, giả sử $2\geq x\geq y\geq z\geq 1$

Xét các tích sau:

  • $\left ( \frac{x}{y}-1 \right )\left ( \frac{y}{z}-1 \right )\geq 0\Rightarrow \frac{x}{y}+\frac{y}{z}\leq \frac{x}{z}+1$
  • $\left ( \frac{y}{x}-1 \right )\left ( \frac{z}{y}-1 \right )\geq 0\Rightarrow \frac{y}{x}+\frac{z}{y}\leq \frac{z}{x}+1$

Vậy:

$F\leq 5+2\left ( \frac{z}{x}+\frac{x}{z} \right )$

Đặt $t=\frac{x}{z}$, dễ chứng minh: $t\in \left [ 1;2 \right ]$ 

Khi đó: $F\leq 5+2\left ( t+\frac{1}{t} \right )$

 

Khảo sát hàm số trên với $t\in \left [ 1;2 \right ]$, ta được $F\leq 10$




#635279 Tổng hợp các bài BĐT trong các đề thi thử THPT Quốc Gia môn Toán năm 2017

Đã gửi bởi phamngochung9a on 24-05-2016 - 21:07 trong Bất đẳng thức và cực trị

Bài 103:cho các số thực dương a,b,c thỏa mãn 9ab+17bc+14ac+12c-18>0 và$a^{2}+b^{2}+c^{2}= 14$.Tìm giá trị nhỏ nhất của biểu thức:

P=$\frac{8(7+ab)\sqrt{5}}{3\sqrt{9ab+17bc+14ac+12c-18}}+\frac{36}{\sqrt{a+b+c+3}}$

Gặp bài này mới thấy rõ được tầm quan trọng của việc lựa chọn điểm rơi....

Từ giả thiết, ta có:

$\left ( a^{2}+b^{2} \right )\left ( 1+4 \right )\geq \left ( a+2b \right )^{2}\\\Rightarrow a^{2}+b^{2}\geq \frac{\left ( a+2b \right )^{2}}{5}\\\Rightarrow 18\geq \frac{\left ( a+2b \right )^{2}}{5}+\frac{5}{9}c^{2}+\frac{4}{9}c^{2}+4\\\geq \frac{2}{3}c\left ( a+2b \right )+\frac{4}{3}c\\\Rightarrow c\left ( a+2b \right )+4c\leq 27\\\Rightarrow 12c\leq 81-3ca-6bc$

Vậy:

$P\geq \frac{4\left [ \left ( a+b \right )^{2}+c^{2} \right ]\sqrt{5}}{3\sqrt{9ab+11bc+11ca+63}}+\frac{36}{\sqrt{a+b+c+3}}\\\geq \frac{2\sqrt{5}\left ( a+b+c \right )^{2}}{3\sqrt{\frac{9}{2}\left ( a+b+c \right )^{2}+2c\left ( a+b \right )}}+\frac{36}{\sqrt{a+b+c+3}}\\\geq \frac{2\sqrt{5}\left ( a+b+c \right )^{2}}{3\sqrt{\frac{9}{2}\left ( a+b+c \right )^{2}+\frac{\left ( a+b+c \right )^{2}}{2}}}+\frac{36}{\sqrt{a+b+c+3}}\\=\frac{2}{3}\left ( a+b+c \right )+\frac{36}{\sqrt{a+b+c+3}}$

Đặt $a+b+c=t$ và xét hàm là ngon roài  :D

Vậy $\min P=16\Leftrightarrow \left\{\begin{matrix} a=1 & & \\ b=2 & & \\ c=3 & & \end{matrix}\right.$